Conditional Probability question

Click For Summary
The discussion revolves around calculating the conditional probability of a student receiving a good grade given that they did not participate in class. It is established that students who participate are four times more likely to achieve a good grade compared to those who do not. The known probabilities include 15% of students participating and 20% achieving good grades. The user attempts to derive the conditional probability but encounters discrepancies in their calculations, leading to confusion about the correct value. Ultimately, the user seeks clarification and assistance in resolving the apparent contradictions in their findings.
Juanriq
Messages
39
Reaction score
0

Homework Statement


Students who actively participate in class are 4 times more likely to get a good grade than those who don't. 15% of students actively participate in class; 20% of students get a good grade. A person did not participate in class, what is the probability that he got a good grade?


Homework Equations





The Attempt at a Solution



Well, I define P(G): student got a good grade; P(P): student participates. Obviously, P(G) = .2 and P(P) = .15. I also know that P(G|P) = 4P(G| not P). I am looking for the conditional probability P(G| not P). I would imagine that I need to find P(G|P) / 4, but this would mean that I need P(G \cap P) P(P) and alas, I do not believe that I have P(G \cap P). ANy ideas? Anything I am missing? Thanks!
 
Physics news on Phys.org
Note that

P(G) = P(G \cap P) + P(G \cap P^c) = P(G|P)P(P) + P(G|P^c) P(P^c)

You know everything except P(G|P) and P(G|P^c), and you know a relation between those two which should allow you to solve for P(G|P).
 
I must be doing something wrong... I have

.2 = (.15)P(G|P) + (.85)P(G|P^c) = (.15)P(G|P^c)/4 + (.85)P(G|P^c)

and upon simplification I get P(G|P^c) = .225, but the answer is supposed to be .16.
 
Here is what I get. Write x = P(G | P^c). Then

0.2 = (0.15) (4x) + (0.85) x = 1.45x

so

x = 0.2 / 1.45 = 0.1379...

which doesn't match either of your answers.

P.S. You know .225 has to be wrong, since P(good grade) = 0.2 and surely the probability shouldn't go up for students who don't participate!
 
Thanks for all the help! It wouldn't be the first typo in the notes...
 
Question: A clock's minute hand has length 4 and its hour hand has length 3. What is the distance between the tips at the moment when it is increasing most rapidly?(Putnam Exam Question) Answer: Making assumption that both the hands moves at constant angular velocities, the answer is ## \sqrt{7} .## But don't you think this assumption is somewhat doubtful and wrong?

Similar threads

Replies
1
Views
1K
Replies
2
Views
1K
  • · Replies 1 ·
Replies
1
Views
2K
  • · Replies 4 ·
Replies
4
Views
2K
  • · Replies 1 ·
Replies
1
Views
1K
  • · Replies 16 ·
Replies
16
Views
2K
  • · Replies 6 ·
Replies
6
Views
2K
  • · Replies 30 ·
2
Replies
30
Views
5K
  • · Replies 1 ·
Replies
1
Views
1K
  • · Replies 6 ·
Replies
6
Views
3K